素数の逆数和が発散することの証明|思考力を鍛える数学

・ 等比級数の和の公式 $$1+r+r^2+\cdots=\frac{1}{1-r}  (0 \le r < 1)$$ ・ 対数関数の性質 $$\log ab=\log a+\log b,\log \frac{1}{a}=-\log a$$ ・ 素因数分解の一意性

・ 対数関数の極限 $$\lim_{n \rightarrow \infty} \log \log n=\infty$$

調和級数が発散すること,すなわち, $$\sum_{n=1}^{\infty} \frac{1}{n}=1+\frac{1}{2}+\frac{1}{3}+\frac{1}{4}+\cdots =\infty$$ が成り立つことはよく知られています.

これは言い換えれば,自然数の逆数和は発散するということです.実はもっと強く,素数の逆数和は発散することが知られています.

$p_n$ を $n$ 番目の素数とします.

$$p_1=2,p_2=3,p_3=5,…$$ です.今回の目標は, $$\sum_{n=1}^{\infty} \frac{1}{p_k}=\infty$$ を示すことです.

素数の逆数和が発散することを証明するための準備として,次の重要な不等式を理解する必要があります.この不等式が証明の中で最も難しい部分となります.

 $p_n$ を $n$ 番目の素数とする.このとき,次の不等式が成り立つ. $$\large \prod_{k=1}^n \frac{1}{1-\frac{1}{p_k}} \ge \sum_{k=1}^{p_n} \frac{1}{k}$$

不等式の意味

右辺は簡単です.これは,$1$ から $p_n$ までの逆数和です. $$\sum_{k=1}^{p_n} \frac{1}{k}=1+\frac{1}{2}+\frac{1}{3}+\cdots+\frac{1}{p_n}$$ 左辺は, $$\frac{1}{1-\frac{1}{p_k}}$$ という式を $k=1$ から $n$ まで掛け合わせたものです.ここで,$\frac{1}{1-\frac{1}{p_k}}$ は,初項 $1$,公比 $\frac{1}{p_k}$ の等比級数の和であることに注意してください.つまり, $$\frac{1}{1-\frac{1}{p_k}}=1+\frac{1}{p_k}+\frac{1}{p_k^2}+\frac{1}{p_k^3}+\cdots$$ です.

不等式の証明

左辺を書き下せば, $$\prod_{k=1}^n \frac{1}{1-\frac{1}{p_k}}=\left(1+\frac{1}{p_1}+\frac{1}{p_1^2}+\cdots\right)\left(1+\frac{1}{p_2}+\frac{1}{p_2^2}+\cdots\right)\cdots\left(1+\frac{1}{p_n}+\frac{1}{p_n^2}+\cdots\right) \cdots(*)$$ という風になります.これを展開したときにどのような項が出てくるかを考えます.$1$ 以上 $p_n$ 以下のいかなる自然数も, $$p_1^{\alpha_1}p_2^{\alpha_2}\cdots p_n^{\alpha_n}$$ という風に素因数分解できます.(ただし,$\alpha_1,…,\alpha_n$ は $0$ 以上の自然数) そして,$(*)$ 式の右辺を展開すれば,$\alpha_1,…,\alpha_n$ が $0$ 以上のどのような自然数だとしても,必ず $$\frac{1}{p_1^{\alpha_1}p_2^{\alpha_2}\cdots p_n^{\alpha_n}}$$ という項がでてきます.これは, $$\prod_{k=1}^n \frac{1}{1-\frac{1}{p_k}} \ge \sum_{k=1}^{p_n} \frac{1}{k}$$ が成り立つことを意味しています.つまり,右辺のどの項も,左辺の式を展開したときに現れるということです.

上で示した不等式を使って,素数の逆数和が発散することを証明します.

証明: $p_n$ を $n$ 番目の素数とすると,次の不等式が成り立つ. $$\prod_{k=1}^n \frac{1}{1-\frac{1}{p_k}} \ge \sum_{k=1}^{p_n} \frac{1}{k}$$ また,$y=\frac{1}{x}$ のグラフより, $$\sum_{k=1}^{p_n} \frac{1}{k} > \int_{1}^{p_n+1} \frac{1}{x} dx=\log (p_n+1) > \log p_n  $$ が成り立つ.→補足
合わせると, $$\prod_{k=1}^n \frac{1}{1-\frac{1}{p_k}} >\log p_n$$ 両辺の逆数をとって, $$\prod_{k=1}^n \left(1-\frac{1}{p_k}\right) < \frac{1}{\log p_n}$$ 両辺どちらも正なので,両辺の log をとることができて, $$\log\left(\prod_{k=1}^n \left(1-\frac{1}{p_k}\right)\right) < \log\left(\frac{1}{\log p_n}\right)$$ したがって, $$\sum_{k=1}^n \log\left(1-\frac{1}{p_k}\right) < -\log \log p_n$$ が成り立つ.ここで,$-\frac{1}{2} \le x < 0$ の範囲で,$2x < \log(1+x)$ が成り立つので,→補足これに $x=-\frac{1}{p_k}$ を代入すると, $$-\frac{2}{p_k} < \log\left(1-\frac{1}{p_k}\right)$$ が従う.

これと上の不等式を合わせると, $$\sum_{k=1}^n -\frac{2}{p_k} < -\log \log p_n$$ よって, $$\sum_{k=1}^n \frac{1}{p_k} > \frac{1}{2}\log \log p_n$$ $n \rightarrow \infty$ とすると,$\frac{1}{2}\log \log p_n \rightarrow \infty$ なので, $$\sum_{k=1}^\infty \frac{1}{p_k} = \infty$$

上の証明中で使った事柄についての補足です.

$\sum_{k=1}^{p_n} \frac{1}{k} > \int_{1}^{p_n+1} \frac{1}{x} dx$ が成り立つこと

x < \log(1+x) (-\frac{1}{2} \le x < 0)$ が成り立つこと

この他にもいろいろな証明が知られています.

Copied title and URL